Fiche de mathématiques
> >

Baccalauréat Asie 2018

Partager :

Bac ES-L (spé et non spé) Asie 2018 : image 7


5 points

exercice 1 Commun à tous les candidats

Bac ES-L (spé et non spé) Asie 2018 : image 9

Bac ES-L (spé et non spé) Asie 2018 : image 12

Bac ES-L (spé et non spé) Asie 2018 : image 11

Bac ES-L (spé et non spé) Asie 2018 : image 10


4 points

exercice 2 Commun à tous les candidats


Bac ES-L (spé et non spé) Asie 2018 : image 6

Bac ES-L (spé et non spé) Asie 2018 : image 1


5 points

exercice 3 Candidats de la série ES n'ayant pas suivi l'enseignement de spécialité et Candidats de la série L

Bac ES-L (spé et non spé) Asie 2018 : image 5

Bac ES-L (spé et non spé) Asie 2018 : image 2


5 points

exercice 3 Candidats de la série ES ayant suivi l'enseignement de spécialité

Bac ES-L (spé et non spé) Asie 2018 : image 3

Bac ES-L (spé et non spé) Asie 2018 : image 4


6 points

exercice 4 Commun à tous les candidats

Bac ES-L (spé et non spé) Asie 2018 : image 8

Bac ES-L (spé et non spé) Asie 2018 : image 13

Bac ES-L (spé et non spé) Asie 2018 : image 14





Bac ES-L (spé et non spé) Asie 2018

Partager :



5 points

exercice 1 - Commun à tous les candidats

{\red{\text{1. }}\blue{\mathbf{Réponse\ d)}\ 0,58}}
Définissons la variable aléatoire X calculant le nombre de réponses reçues.
Nous renouvelons 25 fois de manière indépendante et identique le tirage d'une entreprise.
Lors de chaque tirage, deux issues sont possibles :
le "succès" (l'entreprise a répondu à la personne) de probabilité p = 0,2
l'"échec" (l'entreprise n'a pas répondu à la personne) de probabilité 1 - p = 1 - 0,2 = 0,8.

La variable aléatoire X suit une loi binomiale de paramètres n = 25 et p = 0,2.

Nous devons calculer P(X supegal 5).

P(X\ge5)=1-P(X\le4) \\\phantom{P(X\ge5)}\approx1-0,42\ \ \ \ (\text{à l'aide de la calculatrice}) \\\phantom{P(X\ge5)}\approx0,58 \\\\\Longrightarrow\boxed{P(X\ge5)\approx0,58}
D'où la réponse d) est correcte.

{\red{\text{2. }}\blue{\mathbf{Réponse\ c)}\ P(X<20)=P(X>40)}}
La variable aléatoire X   suit la loi normale d'espérance mu = 30 et d'écart-type sigma.
La courbe de densité de cette loi possède un axe de symétrie d'équation X = mu.

\left\lbrace\begin{matrix}P(X<\mu-10)=P(X>\mu+10)\\\mu=30\ \ \ \ \ \ \ \ \ \ \ \ \ \ \ \ \ \ \ \ \ \ \ \ \ \ \ \ \ \ \ \ \ \end{matrix}\right.\ \ \Longrightarrow P(X<30-10)=P(X>30+10)\\\phantom{..........................................................}\Longrightarrow\boxed{P(X<20)=P(X>40)}
D'où la réponse c) est correcte.

{\red{\text{3. }}\blue{\mathbf{Réponse\ b)}\ 31\%}}
Le taux de variation en pourcentage des ventes entre 2014 et 2016 est donné par

\dfrac{\text{Valeur en 2016}-\text{Valeur en 2014}}{\text{Valeur en 2014}}\times100=\dfrac{4,3-6,2}{6,2}\times100\approx-30,645
Donc les ventes ont diminué d'environ 31 % entre 2014 et 2016.
D'où la réponse b) est correcte.

{\red{\text{4. }}\blue{\mathbf{Réponse\ d)}\ \text{F est décroissante sur }[-3\;-1]}}
Si F  est une primitive de f  sur R, alors F' = f.
La représentation graphique de f  montre que f  est négative sur l'intervalle [-3 ; -1], soit que F'  est négative sur l'intervalle [-3 ; -1].
Si F'  est négative sur l'intervalle [-3 ; -1], alors F  est décroissante sur [-3 ; -1].
D'où la réponse d) est correcte.

{\red{\text{5. }}\blue{\mathbf{Réponse\ d)}}}
Par lecture graphique de la courbe représentative de la fonction f , nous observons que cette fonction f  est convexe sur l'intervalle ]-infini ; -1], concave sur l'intervalle [1 ; 3] et convexe sur l'intervalle [4 ; +infini[.
Dès lors, la dérivée seconde f"  doit être positive sur l'intervalle ]-infini ; -1], négative sur l'intervalle [1 ; 3] et positive sur l'intervalle [4 ; +infini[.
La courbe d) est la seule courbe répondant à ces critères.
D'où la réponse d) est correcte.

4 points

exercice 2 - Commun à tous les candidats

Partie A

Soit X  la variable aléatoire exprimant le nombre de jours utilisés lors de la traversée.
La variable aléatoire X  suit la loi normale de moyenne mu = 7 et d'écart-type sigma = 1.

1.   Nous devons calculer P (5 infegal X  infegal 8).
A l'aide de la calculatrice, nous obtenons  \boxed{P(5\le X \le8)\approx0,819}
Par conséquent, la probabilité que le navigateur termine sa course entre 5 jours et 8 jours après le départ est environ égale à 0,819.

2.   Le record du monde sera battu si le navigateur termine sa course en moins de 5 jours.
Nous devons donc calculer P (X  < 5).

Nous savons que P(X\le\mu)=0,5, soit que P(X\le7)=0,5

Dès lors, P(X\le7)=P(X<5)+P(5\le X\le7)\Longrightarrow 0,5=P(X<5)+P(5\le X\le7)

Or, par la calculatrice, nous avons : P(5\le X\le7)\approx0,47724986

\text{D'où }\ 0,5\approx P(X<5)+0,47724986\Longrightarrow P(X<5)\approx0,5-0,47724986 \\\phantom{\text{D'où }\ 0,5\approx P(X<5)+0,47724986}\Longrightarrow P(X<5)\approx0,02275014 \\\phantom{\text{D'où }\ 0,5\approx P(X<5)+0,47724986}\Longrightarrow \boxed{P(X<5)\approx0,023}
Par conséquent, la probabilité que le navigateur batte le record du monde est environ égale à 0,023.

Partie B

1.   Arbre pondéré représentant la situation.

Bac ES-L (spé et non spé) Asie 2018 : image 15


2.  Nous devons calculer  P(\overline{F}).
En utilisant la formule des probabilités totales, nous avons :
\overset{.}{P(\overline{F})= P(M\cap \overline{F})+P(\overline{M}\cap \overline{F})} \\\phantom{P(\overline{F})}=P(M)\times P_{M}(\overline{F})+P(\overline{M})\times P_{\overline{M}}(\overline{F})\ \\\phantom{P(\overline{F})}=0,16\times0,05+0,84\times0,5 \\\phantom{P(\overline{F})}=0,428 \\\\\Longrightarrow\boxed{P(\overline{F})=0,428}

3.  Nous devons calculer  P_{\overline{F}}(M).
\overset{.}{P_{\overline{F}}(M)=\dfrac{P(M\cap\overline{F})}{P(\overline{F})}=\dfrac{P(M)\times P_M(\overline{F})}{P(\overline{F})}} \\\\\phantom{P_{\overline{F}}(M)}=\dfrac{0,16\times 0,05}{0,428}\approx0,019
Par conséquent, sachant que l'entreprise a finalement choisi de ne pas financer le navigateur, la probabilité que celui-ci ait tout de même réalisé la traversée en moins de 6 jours est environ égale à 0,019.

Partie C

Déterminons un intervalle de fluctuation asymptotique I280  au seuil de 95 % de la proportion de sportifs montés sur le podium dans cet échantillon de 280 sportifs pris au hasard.

Les conditions d'utilisation de l'intervalle de fluctuation sont remplies.
En effet,

\left\lbrace\begin{array}l n=280\ge30 \\ p=0,97\Longrightarrow np=280\times0,97=271,6>5 \\n(1-p)= 280\times(1-0,97)= 280\times0,03=8,4>5 \end{array}

Donc un intervalle de fluctuation asymptotique I280  au seuil de 95% est :

I_{280}=\left[0,97-1,96\sqrt{\dfrac{0,97 (1-0,97)}{280}};0,97+1,96\sqrt{\dfrac{0,97 (1-0,97)}{280}}\right]\\\\\Longrightarrow\boxed{I_{280}\approx[0,950;0,989]}

Une étude des résultats sportifs de l'année a révélé que, parmi 280 sportifs, 263 sont montés sur le podium.
La fréquence observée des sportifs montés sur le podium est  \overset{.}{\boxed{f=\dfrac{263}{280}\approx0,939}}
Nous remarquons que  f\notin I_{280}.
Par conséquent au risque de se tromper de 5%, on ne peut pas adhérer au slogan proposé par l'entreprise.

5 points

exercice 3 - Candidats de la série ES n'ayant pas suivi l'enseignement de                                 spécialité et Candidats de la série L

1. a)   En 2017, le pays compte 300 loups.
Une augmentation de 12% de la population de loups correspond à un coefficient multiplicateur de 1 + 0,12 = 1,12.
Après l'augmentation, le nombre de loups est    1,12\times300=336.
Les chasseurs sont autorisés à tuer 18 loups.
336 - 18 = 318.
Donc, en 2018, le nombre de loups de ce pays s'élève à 318.

1. b)   En 2017 + n , le pays compte un  loups.
Une augmentation de 12% de la population de loups correspond à un coefficient multiplicateur de 1 + 0,12 = 1,12.
Après l'augmentation, le nombre de loups est  1,12\times u_n.
Les chasseurs sont autorisés à tuer 18 loups.
Donc, en 2017 + (n  + 1), le nombre de loups de ce pays s'élève à :  \boxed{u_{n+1}=1,12u_n-18}

2.   Algorithme complété :

            \begin{array}{|c|}\hline N\longleftarrow0\ \ \ \ \ \ \ \ \ \ \ \ \ \ \ \ \ \ \ \ \ \ \ \\U\longleftarrow300\ \ \ \ \ \ \ \ \ \ \ \ \ \ \ \ \ \ \ \ \\\text{Tant que }\ {\red{U<600}}\ \ \text{faire} \\\ \ \ \ |U\longleftarrow{\red{1,12\times U-18}} \\\ \ \ |N\longleftarrow {\red{N+1}}\ \ \ \ \ \ \ \ \ \  \\\text{Fin Tant que}\ \ \ \ \ \ \ \ \ \ \ \ \ \ \ \ \ \\\hline \end{array}

3.   v_n=u_n-150\ \ \ (n\in\mathbb{N})

3. a)   Montrons que la suite (vn ) est géométrique.

v_{n+1}=u_{n+1}-150 \\\phantom{v_{n+1}}=(1,12u_n-18)-150 \\\phantom{v_{n+1}}=1,12u_n-168 \\\phantom{v_{n+1}}=1,12u_n-1,12\times150 \\\phantom{v_{n+1}}=1,12(u_n-150) \\\phantom{v_{n+1}}=1,12v_n \\\\\Longrightarrow\boxed{v_{n+1}=1,12v_n}

D'où la suite (vn ) est une suite géométrique de raison q  = 1,12 et dont le premier terme est  v_0=u_0-150=300-150=150.

3. b)   Le terme général de la suite (vn ) est  v_n=v_0\times q^n , soit  v_n=150\times1,12^n.

\text{Dès lors }\ v_n=u_n-150\Longrightarrow u_n=v_n+150 \\\phantom{\text{Dès lors }\ v_n=u_n-150}\Longrightarrow\overset{.}{\boxed{u_n=150\times1,12^n+150}}

3. c)   Nous savons que  \lim\limits_{n\to+\infty}1,12^n=+\infty\ \ \ \text{car }\ 1,12>1

\text{Donc }\lim\limits_{n\to+\infty}u_n=\lim\limits_{n\to+\infty}(150\times1,12^n+150) \\\\\phantom{\text{Donc }\lim\limits_{n\to+\infty}u_n}=150\times\lim\limits_{n\to+\infty}(1,12^n)+150 \\\phantom{\text{Donc }\lim\limits_{n\to+\infty}u_n}=+\infty \\\\\Longrightarrow\boxed{\lim\limits_{n\to+\infty}u_n=+\infty}

Par conséquent, au fil des années, le nombre de loups va croître indéfiniment.

4. a)   Résoudre dans l'ensemble N l'inéquation : 150 + 1,12n multiplie 150 > 600.

150+1,12^n\times150>600\Longleftrightarrow1,12^n\times150>450 \\\\\phantom{150+1,12^n\times150>600}\Longleftrightarrow 1,12^n>\dfrac{450}{150} \\\\\phantom{150+1,12^n\times150>600}\Longleftrightarrow 1,12^n>3 \\\phantom{150+1,12^n\times150>600}\Longleftrightarrow \ln(1,12^n)>\ln(3) \\\phantom{150+1,12^n\times150>600}\Longleftrightarrow n\times\ln(1,12)>\ln(3) \\\\\phantom{150+1,12^n\times150>600}\Longleftrightarrow n>\dfrac{\ln(3)}{\ln(1,12)}\ \ \ \ \text{(Conservation du sens de l'inégalité car }\ln(1,12)>0) \\\\\text{Or }\ \dfrac{\ln(3)}{\ln(1,12)}\approx9,69

Puisque n est un nombre entier naturel, l'inéquation est vérifiée pour n supegal 10.
Par conséquent, l'ensemble des solutions de l'inéquation est l'ensemble des nombres naturels appartenant à l'intervalle [10 ; +infini[.

4. b)  Dans le contexte de l'énoncé, nous pouvons prévoir que le nombre de loups deviendra supérieur à 600 dès l'année 2017+10, soit dès l'année 2027.
Cela signifie qu'à partir de l'année 2027, le nombre de loups aura doublé.


5.   Une méthode de résolution est, par exemple, de modifier l'algorithme de la question 2) de la manière suivante :

          \begin{array}{|c|}\hline N\longleftarrow0\ \ \ \ \ \ \ \ \ \ \ \ \ \ \ \ \ \ \ \ \ \ \ \\U\longleftarrow{\red{446}}\ \ \ \ \ \ \ \ \ \ \ \ \ \ \ \ \ \ \ \ \\\text{Tant que }\ U<600\ \ \text{faire} \\\ \ \ \ |U\longleftarrow{\red{1,12\times U-35}} \\\ \ \ |N\longleftarrow N+1\ \ \ \ \ \ \ \ \ \  \\\text{Fin Tant que}\ \ \ \ \ \ \ \ \ \ \ \ \ \ \ \ \ \\\hline \end{array}

La valeur de la variable N après l'exécution complète de l'algorithme égale à 7.
2023 + 7 = 2030.
Par conséquent, en 2030, la population de loups dépassera 600 loups.

5 points

exercice 3 - Candidats de la série ES ayant suivi l'enseignement de                                 spécialité et Candidats de la série L

1. a)   Graphe probabiliste complété :

Bac ES-L (spé et non spé) Asie 2018 : image 16


1. b)  La matrice de transition M  du graphe probabiliste dans l'ordre A-B est \boxed{M=\begin{pmatrix}0,7 & 0,3\\0,5 & 0,5\end{pmatrix}}

2. a)   Le premier jour, Lisa se rend au travail à vélo.
             Donc a1 = 1 et b1 = 0.
2. b)  Si pour tout entier naturel n  non nul, nous notons  P_n=\begin{pmatrix}a_n & b_n\end{pmatrix}  la matrice exprimant l'état du nième jour, alors  P_1=\begin{pmatrix}a_1 & b_1\end{pmatrix}=\begin{pmatrix}1 & 0\end{pmatrix}.

Nous devons déterminer la matrice P_8.

P_8=P_1\times M^7\ \ \text{avec }P_1=(1\ \ \ \ \ 0)\\\\\text{Or  }M=\begin{pmatrix}0,7&0,3\\0,5&0,5\end{pmatrix}\Longrightarrow M^7\approx\begin{pmatrix}0,625&0,375\\0,625&0,375\end{pmatrix}\\\\\text{D'où  }P_8=(1\ \ \ \ \ 0)\times\begin{pmatrix}0,625&0,375\\0,625&0,375\end{pmatrix}\\\\\phantom{\text{D'où  }P_8}=(1\times0,625+0\times0,625\ \ \ \ 1\times0,375+0\times0,375)\\\\\phantom{\text{D'où  }P_8}=(0,625\ \ \ \ 0,375)\\\\\Longrightarrow\boxed{P_8=(0,625\ \ \ 0,375)}
Donc la probabilité que Lisa prenne le vélo 8è jour est environ égale à 0,63 (valeur arrondie au centième).

3.   La matrice M  de transition ne comporte pas de 0.
L'état probabiliste Pn  à l'étape n  converge vers un état P  indépendant de l'état initial P1 .
Cet état P  est l'état probabiliste stable du système et vérifie la relation PmultiplieM = P.

Soit  P=\begin{pmatrix}x & y\end{pmatrix}\ \ \ \text{avec }x+y=1

Alors

 P\times M=P

\Longleftrightarrow\begin{pmatrix}x & y\end{pmatrix}\times\begin{pmatrix}0,7&0,3 \\ 0,5 & 0,5\end{pmatrix}=\begin{pmatrix}x & y\end{pmatrix}\ \ \ \text{avec }x+y=1

\Longleftrightarrow\begin{pmatrix}0,7x+0,5y & 0,3x+0,5y\end{pmatrix}=\begin{pmatrix}x & y\end{pmatrix}\ \ \ \text{avec }x+y=1

\Longleftrightarrow\left\lbrace\begin{array}l 0,7x+0,5y=x\\0,3x+0,5y=y\\x+y=1 \end{array}   \Longleftrightarrow\left\lbrace\begin{array}l 0,7x-x+0,5y=0\\0,3x+0,5y-y=0\\x+y=1 \end{array}  \Longleftrightarrow\left\lbrace\begin{array}l -0,3x+0,5y=0\\0,3x-0,5y=0\\x+y=1 \end{array}

\Longleftrightarrow\left\lbrace\begin{array}l0,3x-0,5y=0\\x+y=1 \end{array} \Longleftrightarrow\left\lbrace\begin{array}l0,3x-0,5y=0\\y=1-x \end{array}\Longleftrightarrow\left\lbrace\begin{array}l0,3x-0,5(1-x)=0\\y=1-x \end{array}

\Longleftrightarrow\left\lbrace\begin{array}l0,3x-0,5+0,5x=0\\y=1-x \end{array}\Longleftrightarrow\left\lbrace\begin{array}l0,8x-0,5=0\\y=1-x \end{array}\Longleftrightarrow\left\lbrace\begin{array}l0,8x=0,5\\y=1-x \end{array}

 \Longleftrightarrow\left\lbrace\begin{array}lx=\dfrac{0,5}{0,8}\\\\y=1-x \end{array}\Longleftrightarrow\left\lbrace\begin{array}lx=0,625\\\\y=1-0,625 \end{array}\Longleftrightarrow\boxed{\left\lbrace\begin{array}lx=0,625\\\\y=0,375 \end{array}}

D'où l'état probabiliste stable est   \boxed{P=\begin{pmatrix}0,625 & 0,375\end{pmatrix}}
Nous en déduisons qu'à long terme, la probabilité que Lisa aille au travail à vélo sera proche de 0,625.

4. a)   Pour tout nombre n  entier naturel non nul,  P_{n+1}=P_n\times M.

\text{D'où }\begin{pmatrix}a_{n+1} & b_{n+1}\end{pmatrix}=\begin{pmatrix}a_{n} & b_n\end{pmatrix}\times\begin{pmatrix}0,7&0,3 \\ 0,5 & 0,5\end{pmatrix} \\\\\phantom{\text{D'où }}\begin{pmatrix}a_{n+1} & b_{n+1}\end{pmatrix}=\begin{pmatrix}0,7a_{n}+0,5b_n & 0,3a_n+0,5b_n\end{pmatrix} \\\\\Longrightarrow\boxed{a_{n+1}=0,7a_{n}+0,5b_n}

4. b)   Pour tout nombre n  entier naturel non nul,

\left\lbrace\begin{matrix}a_{n+1}=0,7a_{n}+0,5b_n\\a_{n}+b_n=1\ \ \ \ \ \ \ \ \ \ \ \ \ \end{matrix}\right.\Longleftrightarrow\left\lbrace\begin{matrix}a_{n+1}=0,7a_{n}+0,5b_n\\b_n=1-a_{n}\ \ \ \ \ \ \ \ \ \ \ \ \ \end{matrix}\right. \\\\\Longrightarrow a_{n+1}=0,7a_{n}+0,5(1-a_n) \\\phantom{\Longrightarrow a_{n+1}}=0,7a_{n}+0,5-0,5a_n \\\phantom{\Longrightarrow a_{n+1}}=0,2a_{n}+0,5 \\\\\Longrightarrow\boxed{a_{n+1}=0,2a_{n}+0,5}

5. a)   Algorithme complété :

            \begin{array}{|c|}\hline N\longleftarrow1\ \ \ \ \ \ \ \ \ \ \ \ \ \ \ \ \ \ \ \ \ \ \ \\A\longleftarrow1\ \ \ \ \ \ \ \ \ \ \ \ \ \ \ \ \ \ \ \ \ \ \ \\\text{Tant que }\ {\red{A\ge0,626}}\ \ \text{faire} \\\ \ \ \ |A\longleftarrow{\red{0,2\times A+0,5}} \\\ \ \ |N\longleftarrow {\red{N+1}}\ \ \ \ \ \ \ \ \ \  \\\text{Fin Tant que}\ \ \ \ \ \ \ \ \ \ \ \ \ \ \ \ \ \\\hline \end{array}

5. b)   Le tableau suivant nous indique les premières valeurs prises par la variable N .

            \begin{array}{|c|c|c|c|c|c|c|c|}\hline \text{Valeurs de }n&1&2&3&4&5\\\hline \text{Valeurs de }a_n&1&0,7&0,64&0,628\ {\blue{(>0,626)}}&0,6256\ {\red{(<0,626)}}\\\hline \end{array}

Donc, à la fin de l'exécution de l'algorithme, la variable N  contient la valeur 5.

6 points

exercice 4 - Commun à tous les candidats

Partie A

1.   Par le graphique, nous observons que l'équation f (x ) = 6 admet comme solution x  environegal 12.

            
Bac ES-L (spé et non spé) Asie 2018 : image 17


2. a)   Le coefficient directeur de la droite T  passant par les points A et B est donné par 
            \overset{.}{\dfrac{y_B-y_A}{x_B-x_A}=\dfrac{14,2-7}{2-0}=\dfrac{7,2}{2}=\boxed{3,6}}
Par conséquent, le coefficient directeur de la droite T  est égal à 3,6.

2. b)   Expression de f' (x ).

f'(x)=[(ax+b)\,\text{e}^{-0,2x}]' \\\\\phantom{f'(x)}=(ax+b)'\,\times\text{e}^{-0,2x}+(ax+b)\,\times[\text{e}^{-0,2x}]' \\\\\phantom{f'(x)}=a\,\times\text{e}^{-0,2x}+(ax+b)\,\times(-0,2x)'\,\text{e}^{-0,2x} \\\\\phantom{f'(x)}=a\,\times\text{e}^{-0,2x}-0,2(ax+b)\,\text{e}^{-0,2x} \\\\\phantom{f'(x)}=[a-0,2(ax+b)]\,\text{e}^{-0,2x} \\\\\Longrightarrow\boxed{f'(x)=(-0,2ax+a-0,2b)\,\text{e}^{-0,2x}}


{\red{2.\ \text{c) }}}\ \left\lbrace\begin{matrix}\text{Le coefficient directeur de la droite }T\text{ est égal à 3,6}\\A(0;7)\in C_f\ \ \ \ \ \ \ \ \ \ \ \ \ \ \ \ \ \ \ \ \ \ \ \ \ \ \ \ \ \ \ \ \ \ \ \ \ \ \ \ \ \ \ \ \ \ \ \ \ \ \end{matrix}\right.\Longleftrightarrow\left\lbrace\begin{matrix}f'(0)=3,6\\f(0)=7\ \ \end{matrix}\right. \\\\\Longrightarrow\left\lbrace\begin{matrix}(-0,2a\times0+a-0,2b)\,\text{e}^{-0,2\times0}=3,6\\\ \ \ \ \ \ \ \ \ \ \ \ \ \ (a\times0+b)\,\text{e}^{-0,2\times0}=7\end{matrix}\right. \\\\\Longrightarrow\left\lbrace\begin{matrix}(0+a-0,2b)\,\text{e}^{0}=3,6\\\ \ \ \ \ \ (0+b)\,\text{e}^{0}=7\end{matrix}\right.\Longrightarrow\left\lbrace\begin{matrix}(a-0,2b)\times1=3,6\\\ \ \ \ \ \ \ \ \ b\times1=7\end{matrix}\right. \\\\\Longrightarrow\boxed{\left\lbrace\begin{matrix}a-0,2b=3,6\\\ \ \ \ \ \ \ b=7\end{matrix}\right.}

\\\\\Longrightarrow\left\lbrace\begin{matrix}a-0,2\times7=3,6\\\ \ \ \ \ \ \ \ \ \ \ b=7\end{matrix}\right.\Longrightarrow\left\lbrace\begin{matrix}a-1,4=3,6\\\ \ \ \ \ \ b=7\end{matrix}\right. \Longrightarrow\boxed{\left\lbrace\begin{matrix}a=5\\b=7\end{matrix}\right. }

Partie B

1.   Variations de la fonction f  définie sur [0 ; 25] par  f(x)=(5x+7)\,\text{e}^{-0,2x}.

Dans la question 2. b), nous avons montré que  f'(x)=(-0,2ax+a-0,2b)\,\text{e}^{-0,2x}.

Si a = 5 et b = 7, alors  f'(x)=(-0,2\times5\times x+5-0,2\times7)\,\text{e}^{-0,2x}
\overset{.}{\Longrightarrow\boxed{f'(x)=(-x+3,6)\,\text{e}^{-0,2x}}}

Puisque e-0,2x  > 0 pour tout x  réel, le signe de f' (x ) sera le signe de (-x  + 3,6).

\text{Si }x\in\R,\text{ alors :}-x+3,6\ge0\Longleftrightarrow x\le3,6\\\phantom{\text{Si }x\in\R,\text{ alors :}}-x+3,6\le0\Longleftrightarrow x\ge3,6 \\\\\text{D'où si }x\in[0\,;25],\text{ alors : }{\red{0\le x\le3,6}}\Longrightarrow -x+3,6\ge0 \Longrightarrow {\red{f'(x)\ge0}} \\\phantom{\text{D'où si }x\in[0\,;25],\text{ alors : }}{\red{ x\ge3,6}}\Longrightarrow -x+3,6\le0 \Longrightarrow {\red{f'(x)\le0}}

\underline{\text{Calculs préliminaires }}\\\\\begin{array}{|c|}\hline f(0)=7\\f(3,6)=(5\times3,6+7)\,\text{e}^{-0,2\times3,6}=25\,\text{e}^{-0,72}\approx12,17\\f(25)=(5\times25+7)\,\text{e}^{-0,2\times25}=132\,\text{e}^{-5}\approx0,89\\ \hline \end{array}\\\\\underline{\text{Tableau de signes de }f'(x)\text{ et variations de }f}\\\\\ \ \ \ \begin{array}{|c|ccccc|}\hline &&&&&\\ x&0&&3,6&&25\\&&&&&\\\hline&&&&&\\ f'(x)&&+&0&-&\\&&&&&\\\hline &&&25\,\text{e}^{-0,72}\approx12,17&& \\ f(x)&&\nearrow&&\searrow& \\ &7&&&&132\,\text{e}^{-5}\approx0,89 \\ \hline \end{array}

Par conséquent, f  est strictement croissante sur l'intervalle [0 ; 3,6]
                                     f  est strictement décroissante sur l'intervalle [3,6 ; 25].

2.   Montrons que l'équation f (x ) = 6 admet une unique solution sur l'intervalle [0 ; 25].

La fonction f  est continue et strictement croissante sur l'intervalle [0 ; 3,6].
Puisque f (0) = 7, nous déduisons que f(x) supegal 7 sur l'intervalle [0 ; 3,6].
D'où l'équation f (x ) = 6 n'admet pas de solution sur l'intervalle [0 ; 3,6].

La fonction f  est continue et strictement décroissante sur l'intervalle [3,6 ; 25].
f (3,6) environegal 12,17 > 6 et f (25) environegal 0,89 < 6.
Donc 6 est compris entre f (3,6) et f (25).
D'après le corollaire du théorème des valeurs intermédiaires, l'équation f (x ) = 6 admet une unique solution alpha appartenant à l'intervalle [3,6 ; 25].

Par conséquent, l'équation f (x ) = 6 admet une unique solution alpha appartenant à l'intervalle [0 ; 25].

Par la calculatrice, nous obtenons  \boxed{\alpha\approx12,1}

3.   Soit F  la fonction définie sur [0 ; 25] par  F(x)=(-25x-160)\,\text{e}^{-0,2x}.

Le logiciel de calcul nous indique que  F'(x)=(5x+7)\,\text{e}^{-0,2x} , soit que F' (x ) = f (x ).
Nous en déduisons que la fonction F  est une primitive de la fonction f  sur l'intervalle [0 ; 25].

\text{D'où }\ \int\limits_0^{25}f(x)\,dx=[F(x)]\limits_0^{25}=F(25)-F(0) \\\phantom{\text{D'où }\ \int\limits_0^{25}f(x)\,dx}=(-25\times25-160)\,\text{e}^{-0,2\times25}-(-25\times0-160)\,\text{e}^{-0,2\times0} \\\phantom{\text{D'où }\ \int\limits_0^{25}f(x)\,dx}=-785\,\text{e}^{-5}-(-160)\,\text{e}^{0}=-785\,\text{e}^{-5}+160\times1 \\\phantom{\text{D'où }\ \int\limits_0^{25}f(x)\,dx}=-785\,\text{e}^{-5}+160 \\\\\Longrightarrow \boxed{\int\limits_0^{25}f(x)\,dx=160-785\,\text{e}^{-5}\approx154,711}

Partie C

1.   La fonction f  une continue positive sur l'intervalle [0 ; 25].
\overset{.}{\int\limits_0^{25}f(x)\,dx}  représente l'aire en unité d'aire de la zone hachurée représentant la piscine.
L'énoncé nous indique que l'unité d'aire est le m².
En outre, nous savons, par la question 3. de la Partie B que  \int\limits_0^{25}f(x)\,dx=160-785\,\text{e}^{-5}\approx154,711.
Donc l'aire de la zone hachurée représentant la piscine est égale à  160-785\,\text{e}^{-5}\ \text{m}^2 , soit environ 154,711 m².

2.   Aire d'un rectangle = Longueur multiplie largeur.
Si  \ell  est la largeur du rectangle, alors  160-785\,\text{e}^{-5}=25\times\ell\Longrightarrow\ell=\dfrac{160-785\,\text{e}^{-5}}{25}\approx6,2

D'où la largeur de la piscine rectangulaire sera d'environ 6,2 mètres (valeur arrondie au dixième).

Remarque :
Nous aurions également pu trouver ce résultat en déterminant la valeur moyenne mu de la fonction f  sur l'intervalle [0 ; 25].

\mu=\dfrac{1}{25-0}\int\limits_0^{25}f(x)\,dx=\dfrac{1}{25}(160-785\,\text{e}^{-5})\approx6,2
Publié le
ceci n'est qu'un extrait
Pour visualiser la totalité des cours vous devez vous inscrire / connecter (GRATUIT)
Inscription Gratuite se connecter
Merci à
Hiphigenie
/
malou Webmaster
pour avoir contribué à l'élaboration de cette fiche


Vous devez être membre accéder à ce service...

Pas encore inscrit ?

1 compte par personne, multi-compte interdit !

Ou identifiez-vous :


Rester sur la page

Inscription gratuite

Fiches en rapport

parmi 1674 fiches de maths

Désolé, votre version d'Internet Explorer est plus que périmée ! Merci de le mettre à jour ou de télécharger Firefox ou Google Chrome pour utiliser le site. Votre ordinateur vous remerciera !